Классические теоремы дифференциального исчисления — различия между версиями

Материал из Викиконспекты
Перейти к: навигация, поиск
м (правило Лопиталя для бесконечностей. копипаста из википедии. извините, нет времени :()
(Правило Лопиталя раскрытия неопределенностей)
Строка 147: Строка 147:
 
: <math>\frac{f(x)}{g(x)}=\frac{1-\frac{g(t)}{g(x)}}{1-\frac{f(t)}{f(x)}}\cdot\frac{f'(c)}{g'(c)}</math>.
 
: <math>\frac{f(x)}{g(x)}=\frac{1-\frac{g(t)}{g(x)}}{1-\frac{f(t)}{f(x)}}\cdot\frac{f'(c)}{g'(c)}</math>.
  
Для <math>x</math>, достаточно близких к <math>a</math>, выражение имеет смысл; предел первого множителя правой части равен единице (так как <math>f(t)</math> и <math>g(t)</math> — [[константа|константы]], а <math>f(x)</math> и <math>g(x)</math> стремятся к бесконечности). Значит, этот множитель равен <math>1+\beta</math>, где <math>\beta</math> — бесконечно малая функция при стремлении <math>x</math> к <math>a</math> справа. Выпишем определение этого факта, используя то же значение <math>\varepsilon</math>, что и в определении для <math>\alpha</math>:
+
Для <math>x</math>, достаточно близких к <math>a</math>, выражение имеет смысл; предел первого множителя правой части равен единице (так как <math>f(t)</math> и <math>g(t)</math> — константы, а <math>f(x)</math> и <math>g(x)</math> стремятся к бесконечности). Значит, этот множитель равен <math>1+\beta</math>, где <math>\beta</math> — бесконечно малая функция при стремлении <math>x</math> к <math>a</math> справа. Выпишем определение этого факта, используя то же значение <math>\varepsilon</math>, что и в определении для <math>\alpha</math>:
 
: <math>\forall \varepsilon_{1}>0\, \exists \delta_{2}>0\ : \forall x(0\le x-a<\delta_{2}\Rightarrow \left| \beta(x) \right| <\varepsilon_{1})</math>.
 
: <math>\forall \varepsilon_{1}>0\, \exists \delta_{2}>0\ : \forall x(0\le x-a<\delta_{2}\Rightarrow \left| \beta(x) \right| <\varepsilon_{1})</math>.
  

Версия 23:22, 17 января 2012


Эта статья находится в разработке!

Теорема Ферма о значении производной в экстремальной точке

Определение:
Точки минимума и максимума:
  • Точка [math] x_0 [/math] называется точкой локального минимума, если [math] \forall x \in \dot{O}(x_0) \ f(x) \ge f(x_0) [/math].
  • Точка [math] x_0 [/math] называется точкой локального максимума, если [math] \forall x \in \dot{O}(x_0) \ f(x) \le f(x_0) [/math].


Сами значения [math] f(x_o) [/math] называются соответственно локальным минимумом и локальным максимумом.

Теорема (Ферма):
Пусть [math] f(x) [/math] существует и дифференцируема в [math] O(x_0) [/math], и [math] x_0 [/math] — точка локального экстремума. Тогда [math] f'(x_0) = 0.[/math]
Доказательство:
[math]\triangleright[/math]

Рассмотрим случай, когда [math] x_0 [/math] — точка локального минимума. Случай с локальным максимумом доказывается аналогично.

[math] \frac{\Delta y}{\Delta x} = \frac{f(x_0 + \Delta x) - f(x_0)}{\Delta x}[/math]; рассмотрим [math] \Delta x \approx 0 [/math].

Заметим, что, по определению локального минимума, [math] f(x_0 + \Delta x) - f(x_0) \ge 0 [/math].

Возможны 2 случая для [math] \Delta x [/math]:

  1. [math] \Delta x \lt 0 \Rightarrow \frac{\Delta y}{\Delta x} \le 0 \Rightarrow f'(x_0) \le 0 [/math]
  2. [math] \Delta x \gt 0 \Rightarrow \frac{\Delta y}{\Delta x} \ge 0 \Rightarrow f'(x_0) \ge 0 [/math]
Отсюда, [math] f'(x_0) = 0 [/math].
[math]\triangleleft[/math]

Замечание: обратная теорема не всегда верна, например, [math] y(x) = x^3, y'(0) = 0,[/math] но [math] y(0) [/math] — не экстремум.


Определение:
Корень уравнения [math]f'(x) = 0[/math] называется стационарной точкой.


Теорема Ролля о нулях производной

Теорема (Ролль):
Пусть [math] f(x) [/math] непрерывна на [math] [a; b] [/math], дифференцируема на [math](a, b)[/math] и [math]f(a) = f(b)[/math]. Тогда существует точка [math] c \in (a; b)[/math], такая, что [math] f'(c) = 0[/math].
Доказательство:
[math]\triangleright[/math]

[math] f(x) [/math] непрерывна на [math] [a; b] [/math], значит, у нее на этом отрезке существуют минимум и максимум. Пусть [math] x_1 [/math] — точка минимума, [math] x_2 [/math] — точка максимума.

Рассмотрим 2 случая:

1) Обе точки граничные, то есть [math] x_1, x_2 [/math] находятся на концах отрезка. Тогда, так как [math] f(a) = f(b) [/math], то [math] f_{max}[a; b] = f_{min}[a; b] [/math]. Значит, [math] f(x) [/math] на [math] [a; b] [/math] — константа, то есть [math]\forall c \in (a; b) \ f'(c) = 0[/math]

2) Хотя бы одна из точек [math] x_1, x_2 [/math] не граничная. Пусть это, например, [math] x_1 [/math]. Тогда по теореме Ферма [math] f'(x_1) = 0[/math].
[math]\triangleleft[/math]

Замечание: для непрерывной функции на заданном отрезке ей принимаются все значения между двумя граничными значениями. Такое же свойство выполняется и для ее производной, хотя она может быть уже разрывной.

Теорема Дарбу о промежуточных значениях производной

Теорема (Дарбу):
Пусть [math] f(x) [/math] дифференцируема на [math] [x_1; x_2], A = f'(x_1), B = f'(x_2)[/math]. Тогда [math] \forall D \in [A; B] \ \exists d \in [x_1; x_2]: D = f'(d) [/math]
Доказательство:
[math]\triangleright[/math]

Для определенности считаем, что [math] A \lt B [/math], обратный случай доказывается аналогично.

Рассмотрим вспомогательную функцию [math] g(x) = f(x) - Dx; g'(x) = f'(x) - D [/math]

[math] D \in [A; B] \Rightarrow g'(x_1) \lt 0, g'(x_2) \gt 0 [/math].

По определению производной, [math] g'(x_1) = \frac{g(x_1 + \Delta x) - g(x_1)}{\Delta x} [/math]

При [math] \Delta x \approx 0, \Delta x \gt 0 \ g(x_1 + \Delta x) \lt g(x_1) [/math]

Аналогично рассмотрим [math] g'(x_2) [/math]: при [math] \Delta x \approx 0, \Delta x \lt 0 \ g(x_2 + \Delta x) \lt g(x_2) [/math]

Функция [math] g(x) [/math] — дифференцируема, а значит, также и непрерывна на [math] [x_1, x_2] [/math], поэтому на этом отрезке существуют минимальное и максимальное значения функции. Из двух предыдущих неравенств следует, что минимальное значение достигается не в граничной точке.

Пусть оно достигается в точке [math] d \in (x_1; x_2) [/math], тогда по теореме Ферма в этой точке [math] g'(d) = 0[/math]. Значит, [math] f'(d) = g'(d) + D = D [/math].
[math]\triangleleft[/math]

Формула конечных приращений Лагранжа

Теорема (Лагранж):
Пусть [math] f [/math] непрерывна на [math] [a; b] [/math] и дифференцируема на [math] (a; b) [/math]. Тогда [math] \exists c \in (a; b): [/math] [math] \frac{f(b) - f(a)}{b - a} [/math] [math] = f'(c) [/math]
Доказательство:
[math]\triangleright[/math]

Рассмотрим вспомогательную функцию [math] g(x) = (f(x) - f(a)) - k(x - a), k = [/math] [math] \frac{f(b) - f(a)}{b - a}[/math].

Заметим, что [math] g(a) = g(b) = 0 [/math], значит, по теореме Ролля, [math] \exists c \in (a; b): g'(c) = 0 [/math].

Но [math] g'(x) = f'(x) - k [/math], значит, [math] f'(c) = k = [/math] [math]\frac{f(b) - f(a)}{b - a} [/math]
[math]\triangleleft[/math]

Формула конечных приращений Коши

Теорема (Коши):
Пусть [math] f, g [/math] непрерывны на [math] [a; b] [/math] и дифференцируемы на [math] (a; b) [/math], [math] g'(x) \ne 0\ \forall x \in (a; b)[/math]. Тогда [math] \exists c \in (a; b): [/math] [math] \frac{f(b) - f(a)}{g(b) - g(a)} = \frac{f'(c)}{g'(c)} [/math].
Доказательство:
[math]\triangleright[/math]

Для начала, докажем, что дробь в левой части равенства определена: по теореме Лагранжа, [math] g(b) - g(a) = g'(d)(b - a) [/math] для некоторого [math]d[/math], по условию, правая часть не равна нулю, значит, [math]g(b) - g(a) \ne 0[/math].

Рассмотрим вспомогательную функцию [math] F(x) = f(x) - f(a) - k(g(x) - g(a)), k = [/math] [math]\frac{f(b) - f(a)}{g(b) - g(a)} [/math].

[math] F(a) = F(b) = 0 [/math], значит, по теореме Ролля, [math] \exists c \in (a; b): F'(c) = 0 [/math].

Но [math] F'(x) = f'(x) - kg'(x) [/math], значит

[math] f'(c) = kg'(c) [/math]

[math] \frac{f'(c)}{g'(c)} = k = \frac{f(b) - f(a)}{g(b) - g(a)} [/math]
[math]\triangleleft[/math]

Замечание: при [math]g(x) = x[/math] получаем частный случай формулы Коши — формулу Лагранжа.

Правило Лопиталя раскрытия неопределенностей

Из формулы Коши можно получить раскрытие неопределенностей вида [math] \frac{0}{0} [/math], [math] \frac{\infty}{\infty} [/math](в числителе и знаменателе дроби получаются нулевые или бесконечные значения). Это правило называют правилом Лопиталя:

Теорема (правило Лопиталя):
Если при [math]x \rightarrow a[/math] [math]\frac{f(x)}{g(x)} = \frac{0}{0} [/math], то [math] \lim\limits_{x \rightarrow a} \frac{f(x)}{g(x)} = \lim\limits_{x \rightarrow a} \frac{f'(x)}{g'(x)} [/math]
Доказательство:
[math]\triangleright[/math]

Доопределим по непрерывности значения функций в точке [math] a [/math]: [math] f(a) = g(a) = 0 [/math].

По формуле Коши для малого отрезка [math] [a; x] [/math] выполняется равенство [math] \frac{f(x) - f(a)}{g(x) - g(a)} = \frac{f'(x)}{g'(x)} [/math].

Подставляя туда [math] f(a), g(a) [/math], получаем требуемое равенство.

Случай с неопределенностью вида [math] \frac{\infty}{\infty} [/math] доказывается аналогично. Докажем теорему для неопределённостей вида [math]\left(\frac{\infty}{\infty}\right)[/math].

Пусть, для начала, предел отношения производных конечен и равен [math]A[/math]. Тогда, при стремлении [math]x[/math] к [math]a[/math] справа, это отношение можно записать как [math]A+\alpha[/math], где [math]\alpha[/math] — O(1). Запишем это условие:

[math]\forall\varepsilon_{1}\gt 0\, \exists \delta_{1}\gt 0 : \forall x(0\le x-a\lt \delta_{1}\Rightarrow \left| \alpha(x)\right| \lt \varepsilon_{1})[/math].

Зафиксируем [math]t[/math] из отрезка [math][a,\;a+\delta_1][/math] и применим теорему Коши ко всем [math]x[/math] из отрезка [math][a,\;t][/math]:

[math]\forall x\in [a;t]\ \exists c\in [a;\;x]\!:\frac{f(x)-f(t)}{g(x)-g(t)}=\frac{f'(c)}{g'(c)}[/math], что можно привести к следующему виду:
[math]\frac{f(x)}{g(x)}=\frac{1-\frac{g(t)}{g(x)}}{1-\frac{f(t)}{f(x)}}\cdot\frac{f'(c)}{g'(c)}[/math].

Для [math]x[/math], достаточно близких к [math]a[/math], выражение имеет смысл; предел первого множителя правой части равен единице (так как [math]f(t)[/math] и [math]g(t)[/math] — константы, а [math]f(x)[/math] и [math]g(x)[/math] стремятся к бесконечности). Значит, этот множитель равен [math]1+\beta[/math], где [math]\beta[/math] — бесконечно малая функция при стремлении [math]x[/math] к [math]a[/math] справа. Выпишем определение этого факта, используя то же значение [math]\varepsilon[/math], что и в определении для [math]\alpha[/math]:

[math]\forall \varepsilon_{1}\gt 0\, \exists \delta_{2}\gt 0\ : \forall x(0\le x-a\lt \delta_{2}\Rightarrow \left| \beta(x) \right| \lt \varepsilon_{1})[/math].

Получили, что отношение функций представимо в виде [math](1+\beta)(A+\alpha)[/math], и [math]\left|\frac{f(x)}{g(x)}-A\right|\lt |A|\varepsilon_{1}+\varepsilon_{1}+\varepsilon_{1}^{2}[/math]. По любому данному [math]\varepsilon[/math] можно найти такое [math]\varepsilon_{1}[/math], чтобы модуль разности отношения функций и [math]A[/math] был меньше [math]\varepsilon[/math], значит, предел отношения функций действительно равен [math]A[/math].

Если же предел [math]A[/math] бесконечен (допустим, он равен плюс бесконечности), то

[math]\forall M\gt 0\, \exists \delta_{1}\gt 0 : \forall x(0\le x-a\lt \delta_{1}\Rightarrow\frac{f'(x)}{g'(x)}\gt 2M)[/math].
В определении [math]\beta[/math] будем брать [math]\varepsilon_{1} \lt \frac{1}{2}[/math]; первый множитель правой части будет больше 1/2 при [math]x[/math], достаточно близких к [math]a[/math], а тогда [math]\frac{f(x)}{g(x)}\gt \frac{1}{2}\cdot 2M=M\Rightarrow \lim_{x\to a+}{\frac{f(x)}{g(x)}}=+\infty[/math].
[math]\triangleleft[/math]